Diễn Đàn MathScopeDiễn Đàn MathScope
  Diễn Đàn MathScope
Ghi Danh Hỏi/Ðáp Community Lịch

Go Back   Diễn Đàn MathScope > Sơ Cấp > Việt Nam và IMO > 2012

News & Announcements

Ngoài một số quy định đã được nêu trong phần Quy định của Ghi Danh , mọi người tranh thủ bỏ ra 5 phút để đọc thêm một số Quy định sau để khỏi bị treo nick ở MathScope nhé !

* Nội quy MathScope.Org

* Một số quy định chung !

* Quy định về việc viết bài trong diễn đàn MathScope

* Nếu bạn muốn gia nhập đội ngũ BQT thì vui lòng tham gia tại đây

* Những câu hỏi thường gặp

* Về việc viết bài trong Box Đại học và Sau đại học


Trả lời Gởi Ðề Tài Mới
 
Ðiều Chỉnh Xếp Bài
Old 02-03-2012, 12:16 AM   #61
namdung
Administrator

 
Tham gia ngày: Feb 2009
Đến từ: Tp Hồ Chí Minh
Bài gởi: 1,343
Thanks: 209
Thanked 4,066 Times in 778 Posts
Gửi tin nhắn qua Yahoo chát tới namdung
Trích:
Nguyên văn bởi leviethai View Post
Mặc dù không được thi nhưng cũng góp vui . Trình độ có hạn, xin giải hai bài BĐT để mọi người tham khảo vậy.
Rất tiếc cho Việt Hải là vòng 1 không có bất đẳng thức, nếu không thì bạn đã có mặt trong vòng TST.

Trích:
Nguyên văn bởi leviethai View Post

Bài 14.Tất nhiên là $n\ge 2. $ Không mất tính tổng quát, giả sử $a^2_n $ lớn nhất.
Chỗ này Hải xem lại, vai trò của các $a_i $ trong bài này là khác nhau.

Trích:
Nguyên văn bởi leviethai View Post

Bài 15. Ta giả sử điều phải chứng minh là sai, tức là ta sẽ có đồng thời 3 bất đẳng thức sau
$\[\frac{{{{(x + y)}^2}}}{{xy}} < \frac{2}{{x + y + z - 1}},\;\frac{{{{(y + z)}^2}}}{{yz}} < \frac{2}{{x + y + z - 1}},\;\frac{{{{(z + x)}^2}}}{{zx}} < \frac{2}{{x + y + z - 1}}.\] $

Suy ra
$\[(x + y + z - 1)\left[ {{{(x + y)}^2} + {{(y + z)}^2} + {{(z + x)}^2}} \right] < 2(xy + yz + zx),\] $

Tương đương với
$\[2(x + y + z)\left[ {{{(x + y)}^2} + {{(y + z)}^2} + {{(z + x)}^2}} \right] < \left[ {{{(x + y)}^2} + {{(y + z)}^2} + {{(z + x)}^2}} \right] + 2(xy + yz + zx),\] $

Tức là
$\[2(x + y + z)({x^2} + {y^2} + {z^2} + xy + yz + zx) < 2{(x + y + z)^2}.\] $

Vậy ta phải có
$\[{x^2} + {y^2} + {z^2} + xy + yz + zx < x + y + z.\] $

Ta có đẳng thức quen thuộc sau
$x^2+y^2+z^2+2xyz=1, $ suy ra $x^2+y^2+z^2=1-2xyz. $

Như vậy bất đẳng thức trên sẽ trở thành
$1-2xyz+xy+yz+zx<x+y+z, $

Hay
$(1-x)(1-y)(1-z)<xyz. $

Thay $x = \frac{{{b^2} + {c^2} - {a^2}}}{{2bc}},\;y = \frac{{{c^2} + {a^2} - {b^2}}}{{2ca}}, $ và $z = \frac{{{a^2} + {b^2} - {c^2}}}{{2ab}}, $ với $a,\;b,\;c $ là độ dài ba cạnh tam giác $ABC. $ Khi đó bằng một chút biến đổi, bất đẳng thức trên trở thành

$\[{(a + b - c)^2}{(b + c - a)^2}{(c + a - b)^2} < ({a^2} + {b^2} - {c^2})({b^2} + {c^2} - {a^2})({c^2} + {a^2} - {b^2}).\] $

Tuy nhiên bất đẳng thức này sai. Do tam giác $ABC $ nhọn nên ${a^2} + {b^2} - {c^2},\;{b^2} + {c^2} - {a^2} $ và ${c^2} + {a^2} - {b^2} $ đều dương, ta xét bất đẳng thức sau

$\[{(a + b - c)^2}{(b + c - a)^2} \ge ({a^2} + {b^2} - {c^2})({b^2} + {c^2} - {a^2}).\] $

Tương đương với
$\[{\left[ {{b^2} - {{(c - a)}^2}} \right]^2} \ge {b^4} - {({c^2} - {a^2})^2}.\] $

Hay
$\[{({c^2} - {a^2})^2} + {(c - a)^4} \ge 2{b^2}{(c - a)^2}.\] $

Bất đẳng thức này đúng vì
$\[{({c^2} - {a^2})^2} + {(c - a)^4} = {(c - a)^2}\left[ {{{(c + a)}^2} + {{(c - a)}^2}} \right] = 2{(c - a)^2}({c^2} + {a^2}) \ge 2{b^2}{(c - a)^2}.\] $

Như vậy, ta sẽ có ba bất đẳng thức sau
$\[{(a + b - c)^2}{(b + c - a)^2} \ge ({a^2} + {b^2} - {c^2})({b^2} + {c^2} - {a^2}),\] $
$\[{(b + c - a)^2}{(c + a - b)^2} \ge ({b^2} + {c^2} - {a^2})({c^2} + {a^2} - {b^2}),\] $
$\[{(c + a - b)^2}{(a + b - c)^2} \ge ({c^2} + {a^2} - {b^2})({a^2} + {b^2} - {c^2}).\] $

Từ đây ta dễ dàng suy ra mâu thuẫn.

Tóm lại, điều giả sử của ta là sai, như vậy ta có điều phải chứng minh. Đẳng thức xảy ra khi và chỉ khi tam giác $ABC $ đều. $\blacksquare $
Bài này thì quá tuyệt vời.
[RIGHT][I][B]Nguồn: MathScope.ORG[/B][/I][/RIGHT]
 
namdung is offline   Trả Lời Với Trích Dẫn
The Following 4 Users Say Thank You to namdung For This Useful Post:
F7T7 (02-03-2012), hoduckhanhgx (06-03-2012), mathscope_me (06-03-2012), pco (03-03-2012)
Old 02-03-2012, 12:37 AM   #62
kien10a1
+Thành Viên+
 
kien10a1's Avatar
 
Tham gia ngày: Feb 2011
Đến từ: Vĩnh Yên- Vĩnh Phúc
Bài gởi: 371
Thanks: 43
Thanked 263 Times in 153 Posts
Gửi tin nhắn qua Yahoo chát tới kien10a1
Bài 20 : Chứng minh rằng có thể phân hoạch tập 1,2,3...1989 thành 117 tập con rời nhau, mỗi tập có 17 phần tử và tổng các phần tử ở mỗi tập là như nhau.

Bài 19: Giả sử I là trực tâm$ A_1B_1C_1 $, có $B_1C_1\perp AI $, biến đổi góc được $B_1,C_1,B,C $ đồng viên, tương tự và suy ra được $A_2, B_2,C_2 $ lần lượt là tâm $(BCB_1C_1),(ABA_1B_1),(CAC_1A_1) $.
Do vậy O là tâm $(ABC) $ thì $OA_2\perp BC, OB_2\perp AC $, dẫn đến $\widehat{OA_2B_2}=\widehat{ICB}=\widehat{ICA}=\wid ehat{OB_2A} $ nên O cách đều $A_2,B_2 $, tương tự có O là tâm $(A_2B_2C_2) $
Chiều ngược lại, các đường thẳng qua $A_2 $, vuông góc BC, qua $B_2 $, vuông góc AC, qua $C_2 $, vuông góc AB đồng quy tại O là tâm cả 2 tam giác ABC và$ A_2B_2C_2 $, suy ra $A_2 $ thuộc trung trực BC, suy ra $BCB_1C_1 $ nội tiếp, tương tự...
[RIGHT][I][B]Nguồn: MathScope.ORG[/B][/I][/RIGHT]
 
__________________
Quay về với nơi bắt đầu

thay đổi nội dung bởi: kien10a1, 02-03-2012 lúc 12:52 AM
kien10a1 is offline   Trả Lời Với Trích Dẫn
The Following User Says Thank You to kien10a1 For This Useful Post:
huynhcongbang (02-03-2012)
Old 02-03-2012, 03:15 PM   #63
huynhcongbang
Administrator

 
huynhcongbang's Avatar
 
Tham gia ngày: Feb 2009
Đến từ: Ho Chi Minh City
Bài gởi: 2,413
Thanks: 2,165
Thanked 4,188 Times in 1,381 Posts
Gửi tin nhắn qua Yahoo chát tới huynhcongbang
Ủng hộ tiếp hai bài tổ hợp:


Bài 21.
Cho $n $ là hai số nguyên dương và xét một bảng có kích thước $n\times n $. Người ta tô màu ít nhất $2n $ ô vuông của bảng. Chứng minh rằng tồn tại hai giá trị $k,l $ sao cho:
i) $1 \le l<k\le n. $
ii) Có thể tô màu $2k $ ô vuông của bảng, giả sử là ${{a}_{1}},{{a}_{2}},{{a}_{3}},...,{{a}_{2k}} $ sao cho ${{a}_{1}},{{a}_{2}} $ thuộc cùng một dòng; ${{a}_{2}},{{a}_{3}} $ thuộc cùng một cột; ${{a}_{3}},{{a}_{4}} $ thuộc cùng một dòng;...;${{a}_{2k-l-1}},{{a}_{2k-l}} $ thuộc cùng một cột; ${{a}_{2k-l}},{{a}_{2k}} $ thuộc cùng một dòng và ${{a}_{2k}},{{a}_{1}} $ thuộc cùng một cột.

Bài 22.
Xét tập hợp S gồm $n $ số nguyên dương đầu tiên. Một tập con T của S được gọi là “tập tốt” nếu với $a,b $ bất kì thuộc T và trung bình cộng của chúng là số nguyên thì trung bình cộng đó cũng thuộc T.
Các tập hợp có ít hơn 2 phần tử cũng được xem là tập tốt. Gọi $A(n) $ là số tập tốt ứng với tập S có n phần tử.
Tìm tất cả các giá trị n sao cho
$A(n+2)-2A(n+1)+A(n)=1. $


--------------------------------


Trích:
Nguyên văn bởi kien10a1 View Post
Bài 20 : Chứng minh rằng có thể phân hoạch tập 1,2,3...1989 thành 117 tập con rời nhau, mỗi tập có 17 phần tử và tổng các phần tử ở mỗi tập là như nhau.
Bài này chính là bài 1, IMO 1989.
http://www.artofproblemsolving.com/F...b57845#p372257
[RIGHT][I][B]Nguồn: MathScope.ORG[/B][/I][/RIGHT]
 
__________________
Sự im lặng của bầy mèo

thay đổi nội dung bởi: huynhcongbang, 02-03-2012 lúc 03:32 PM
huynhcongbang is offline   Trả Lời Với Trích Dẫn
Old 02-03-2012, 04:09 PM   #64
namdung
Administrator

 
Tham gia ngày: Feb 2009
Đến từ: Tp Hồ Chí Minh
Bài gởi: 1,343
Thanks: 209
Thanked 4,066 Times in 778 Posts
Gửi tin nhắn qua Yahoo chát tới namdung
Còn bài 4 chưa có ai giải nè:

4. (Canadian MOCP 2010, trung bình) Cho tam giác ABC có $A > 90^0 $, $AB < AC $, O là tâm đường tròn ngoại tiếp. M, N là trung điểm của BC và AO và D là giao điểm của MN và AC. Biết rằng $AD = \frac{1}{2}(AB+AC) $. Hãy tìm độ lớn góc A.
[RIGHT][I][B]Nguồn: MathScope.ORG[/B][/I][/RIGHT]
 
namdung is offline   Trả Lời Với Trích Dẫn
Old 02-03-2012, 09:28 PM   #65
kien10a1
+Thành Viên+
 
kien10a1's Avatar
 
Tham gia ngày: Feb 2011
Đến từ: Vĩnh Yên- Vĩnh Phúc
Bài gởi: 371
Thanks: 43
Thanked 263 Times in 153 Posts
Gửi tin nhắn qua Yahoo chát tới kien10a1
Bài 4:
Lấy B' đối xứng B qua phân giác góc A thì D là trung điểm CB' nên BB'//MD hay MD vuông góc phân giác góc A, K là điểm chính giữa cung nhỏ BC thì suy ra KA//MN, suy ra M là trung điểm KO nên KBOC là hình thoi, tính được ngay $\widehat{BKC}=\widehat{A}=120 $
[RIGHT][I][B]Nguồn: MathScope.ORG[/B][/I][/RIGHT]
 
__________________
Quay về với nơi bắt đầu
kien10a1 is offline   Trả Lời Với Trích Dẫn
Old 02-03-2012, 09:37 PM   #66
Joe Dalton
+Thành Viên+
 
Joe Dalton's Avatar
 
Tham gia ngày: Jun 2011
Bài gởi: 52
Thanks: 2
Thanked 28 Times in 22 Posts
Trích:
Nguyên văn bởi kien10a1 View Post
Bài 4:
Lấy B' đối xứng B qua phân giác góc A thì D là trung điểm CB' nên BB'//MD hay MD vuông góc phân giác góc A, K là điểm chính giữa cung nhỏ BC thì suy ra KA//MN, suy ra M là trung điểm KO nên KBOC là hình thoi, tính được ngay $\widehat{BKC}=\widehat{A}=120 $
Còn trường hợp $D$ nằm ngoài đoạn $AC$? Khi đó $D$ đối xứng với trung điểm $B'C$ qua $A$.
[RIGHT][I][B]Nguồn: MathScope.ORG[/B][/I][/RIGHT]
 
Joe Dalton is offline   Trả Lời Với Trích Dẫn
Old 02-03-2012, 10:39 PM   #67
kien10a1
+Thành Viên+
 
kien10a1's Avatar
 
Tham gia ngày: Feb 2011
Đến từ: Vĩnh Yên- Vĩnh Phúc
Bài gởi: 371
Thanks: 43
Thanked 263 Times in 153 Posts
Gửi tin nhắn qua Yahoo chát tới kien10a1
Trích:
Nguyên văn bởi Joe Dalton View Post
Còn trường hợp $D$ nằm ngoài đoạn $AC$? Khi đó $D$ đối xứng với trung điểm $B'C$ qua $A$.
Đúng là trường hợp đấy thì không dùng được cách này, mà có lẽ đề là MN cắt cạnh AC, chứ không thì...

Bài 22: Gọi S(n) là số tập tốt chứa n, và là con của {1,2...,n}, ta có S(n+2)=A(n+2)-A(n+1); S(n+1)=A(n+1)-A(n); vậy cần có S(n+2)-S(n+1)=1, mặt khác với mỗi tập tốt chứa n+1 của {1,2...n+1}, ta tăng mỗi phần tử lên 1 đơn vị được tập tốt chứa n+2 của {1,2...n+2}, và tập {1,2...,n+2} cũng chính là 1 tập tốt, vậy có S(n+2)-S(n+1)=1 tương đương tập tốt chứa 1,n+2 phải là tập {1,2,...,n+2}, ta tìm n thỏa mãn điều này là xong.
Trước tiên xét tập A={1,n+2}, ta thực hiện như sau: nếu có 2 phần tử của A có trung bình cộng nguyên thì ta bổ sung số đó vào A, và để thỏa mãn đk ở trên thì quá trình bổ sung sẽ kết thúc khi A=S;ta cũng bổ sung theo nguyên tắc: từ x phân tử đầu ta thêm tất cả số có thể( đợt trước) rồi mới thêm tiếp với các số tạo ra từ các số thứ x+1,... và 1 số nào đó( đợt sau).
Nhận xét, cách bổ sung này đảm bảo nếu không dùng 1, n+2 thì các số bổ sung ở " đợt sau" đều đã có trong tập.
từ đây, ta chỉ xét phần tử nhỏ nhất ở mỗi lần bổ sung
Phải có n+2 lẻ, bổ sung $\frac{1+n+2}{2} $, thì thấy ngay đây lại phải là 1 số lẻ, tương tự
suy ra $1 + \frac{1+n+2}{2} $ lại là 1 số lẻ, và ta lại bổ sung $\frac{1+(1 + \frac{1+n+2}{2})}{2} $, quá trình sẽ dừng nếu ta gặp 1 số chẵn là phần tử bổ sung nhỏ nhất, và để thỏa mãn ycbt thì số cuối cùng phải là 2, làm ngược lại... suy ra n+2 có dạng $2^t+1 $
[RIGHT][I][B]Nguồn: MathScope.ORG[/B][/I][/RIGHT]
 
__________________
Quay về với nơi bắt đầu

thay đổi nội dung bởi: kien10a1, 02-03-2012 lúc 11:42 PM
kien10a1 is offline   Trả Lời Với Trích Dẫn
The Following User Says Thank You to kien10a1 For This Useful Post:
huynhcongbang (03-03-2012)
Old 03-03-2012, 09:48 AM   #68
huynhcongbang
Administrator

 
huynhcongbang's Avatar
 
Tham gia ngày: Feb 2009
Đến từ: Ho Chi Minh City
Bài gởi: 2,413
Thanks: 2,165
Thanked 4,188 Times in 1,381 Posts
Gửi tin nhắn qua Yahoo chát tới huynhcongbang
Kết quả $n+2=2^t+1 \Leftrightarrow n = 2^t -1 $ của em là đúng rồi đấy. Để anh xem lại từ từ!


[RIGHT][I][B]Nguồn: MathScope.ORG[/B][/I][/RIGHT]
 
__________________
Sự im lặng của bầy mèo

thay đổi nội dung bởi: sang89, 03-03-2012 lúc 11:34 AM
huynhcongbang is offline   Trả Lời Với Trích Dẫn
Old 03-03-2012, 12:25 PM   #69
vinvin
+Thành Viên+
 
Tham gia ngày: Sep 2011
Bài gởi: 26
Thanks: 0
Thanked 12 Times in 9 Posts
Trích:
Nguyên văn bởi huynhcongbang View Post
Ủng hộ tiếp hai bài tổ hợp:


Bài 21.
Cho $n $ là hai số nguyên dương và xét một bảng có kích thước $n\times n $. Người ta tô màu ít nhất $2n $ ô vuông của bảng. Chứng minh rằng tồn tại hai giá trị $k,l $ sao cho:
i) $1 \le l<k\le n. $
ii) Có thể tô màu $2k $ ô vuông của bảng, giả sử là ${{a}_{1}},{{a}_{2}},{{a}_{3}},...,{{a}_{2k}} $ sao cho ${{a}_{1}},{{a}_{2}} $ thuộc cùng một dòng; ${{a}_{2}},{{a}_{3}} $ thuộc cùng một cột; ${{a}_{3}},{{a}_{4}} $ thuộc cùng một dòng;...;${{a}_{2k-l-1}},{{a}_{2k-l}} $ thuộc cùng một cột; ${{a}_{2k-l}},{{a}_{2k}} $ thuộc cùng một dòng và ${{a}_{2k}},{{a}_{1}} $ thuộc cùng một cột.
Bài này ta chứng minh quy nạp theo $ n $ với $ n=2 $ thì hiển nhiên đúng.
Giả sử là bài toán đúng với mọi số bé hơn n. Xét hình nxn:
+Nếu có 1 hàng và 1 cột tổng số ô được tô màu trên chúng bé hơn 2 thì bỏ hàng và cột đó đi ta đc hình (n-1)x(n-1) và có ít nhất 2n-2 ô được tô màu. Theo giả thiết quy nạp ta chọn được k,l thỏa đề bài.
+Bây giờ ta xét trường hợp mọi cột đều có ít nhất 2 ô được tô màu( tương tự với hàng) và có ít nhất một hàng i nào đó chỉ chứ 1 ô được tô màu. Khi ta bỏ hàng đó đi thì nếu có một cột j nào đó chỉ chứa 1 ô thì tổng sô ô được tô màu ở hàng i và cột j là 2, quay về trường hợp 1. Còn nếu ko có cột nào có ít hơn 2 ô được tô màu thì bỏ đi 1 cột bất kì ta vẫn được hình (n-1)x(n-1) và có ít nhất 2n-2 ô được tô màu.
+ Cuối cùng là trường hợp mỗi hàng và mỗi cột đều có ít nhất 2 ô được tô màu. Khi đó ta lấy 1 ô bất kì làm xuất phát nối nó vs một ô cùng hàng rồi từ ô đó nối với 1 ô cùng cột, liên tiếp thực hiện các thao tác lần thứ 2n thì nối ở điểm đang đứng với ô cùng cột, thao tác thứ 2n+1 thì nối với ô cùng hàng, ta có chú ý là vì mỗi hàng và cột đều có ít nhất 2 ô nên lúc nào ta cũng thực hiện được thao tác và đến một lúc nào đó những đường nối của ta là khép kín, Khi đấy sẽ chọn được k và l thỏa đề bài.
[RIGHT][I][B]Nguồn: MathScope.ORG[/B][/I][/RIGHT]
 
vinvin is offline   Trả Lời Với Trích Dẫn
The Following 2 Users Say Thank You to vinvin For This Useful Post:
batigoal (03-03-2012), huynhcongbang (06-03-2012)
Old 04-03-2012, 12:46 AM   #70
kien10a1
+Thành Viên+
 
kien10a1's Avatar
 
Tham gia ngày: Feb 2011
Đến từ: Vĩnh Yên- Vĩnh Phúc
Bài gởi: 371
Thanks: 43
Thanked 263 Times in 153 Posts
Gửi tin nhắn qua Yahoo chát tới kien10a1
Bài 23: Cho x,y,z dương tích bằng 1. CMR:
$9+ \frac{1}{x^3}+\frac{1}{y^3}+\frac{1}{z^3}\geq (x+y+z)(xy+yz+zx)+ \frac{(xy+yz+zx)^2}{x+y+z} $

[RIGHT][I][B]Nguồn: MathScope.ORG[/B][/I][/RIGHT]
 
__________________
Quay về với nơi bắt đầu
kien10a1 is offline   Trả Lời Với Trích Dẫn
The Following User Says Thank You to kien10a1 For This Useful Post:
namdung (05-03-2012)
Old 05-03-2012, 09:15 PM   #71
namdung
Administrator

 
Tham gia ngày: Feb 2009
Đến từ: Tp Hồ Chí Minh
Bài gởi: 1,343
Thanks: 209
Thanked 4,066 Times in 778 Posts
Gửi tin nhắn qua Yahoo chát tới namdung
Bài 24. Tìm tất cả các hàm số $f:\; \mathbb R^+ \to\mathbb R^+$ sao cho với mọi $x > 0$ và với mọi $z:\; 0 < z < 1$ ta có:\[ (1-z)f(x) = f\left( \dfrac{(1-z)f(xz)}{z}\right)\]
[RIGHT][I][B]Nguồn: MathScope.ORG[/B][/I][/RIGHT]
 
namdung is offline   Trả Lời Với Trích Dẫn
Old 06-03-2012, 02:23 PM   #72
amateur
Super Moderator
 
Tham gia ngày: Jan 2011
Bài gởi: 17
Thanks: 3
Thanked 4 Times in 3 Posts
Bài 25(Japan 2012): Cho một quân cờ nằm tại gốc tọa độ trên mặt phẳng tọa độ, hai người $A, B$ chơi 1 trò chơi như sau:
  • Đầu tiên, $A$ đánh dấu vào một điểm trên lưới tọa độ, khác với điểm đang đặt quân cờ.
  • Sau đó $B$ thực hiện một bước di chuyển quân cờ từ điểm $(x,y)$ đến điểm $(x+1;y)$ hoặc $(x;y+1)$ $m$ lần với $1\leq m\leq k$ nhưng không được di chuyển quân cờ đến các điểm đánh dấu.
  • Tiếp đến $A$ lại đánh dấu một điểm trên lưới toạ độ. Và trò chơi cứ tiếp tục như thế.
$A$ sẽ chiến thắng nếu $B$ không còn di chuyển được quân cờ. Tìm tất cả các số nguyên dương $k$ sao cho $A$ có thể chiến thắng sau một số hữu hạn lượt đi bất kể $B$ di chuyển quân cờ như thế nào.

Bài này nếu đề được hiểu là sau một lượt đánh dấu của A, B chọn một trong 2 hướng sang phải hoặc lên trên, và đi theo hướng đã chọn từ 1-m bước tùy ý thì đã có ở http://math.vn/showthread.php?17543-...012-B%C3%A0i-5.
Nhưng có người hiểu rằng sau một lượt của A, B có thể đi tùy ý 1-m bước và có thể đi zích zắc, tức là có thể đi theo 2 hướng bao nhiêu bước tùy ý trong vòng <=m bước. Mời các bạn cùng thảo luận. .

Bài 26:Một con đang đứng ở ô $(i,j)$ của bàn cờ thì nó có thể đi đến các ô $(i \pm 1,j \pm 2)$ hoặc $(i \pm 2,j \pm 1)$ (như cách đi trong cờ vua hoặc cờ tướng).
  1. Một bàn cờ kích thước $2011 \times 2011$ bị khoét đi một hình vuông cỡ $2005 \times 2005$ ở trung tâm bàn cờ. Một con đứng ở ô $(1,1)$, chứng minh rằng con có thể đi hết tất cả các ô, mỗi ô chỉ qua đúng 1 lần.
  2. Con đứng ở ô $(1,1)$ của bàn cờ cỡ $2010 \times 2012 $, nó chỉ có thể thoát khỏi bàn cờ nếu trên bàn cờ có ít hơn $2010$ ô mà nó chưa đi qua. Hỏi con có thoát khỏi bàn cờ được hay không ?. (Mỗi ô chỉ được qua 1 lần)

[RIGHT][I][B]Nguồn: MathScope.ORG[/B][/I][/RIGHT]
 

thay đổi nội dung bởi: amateur, 06-03-2012 lúc 02:27 PM
amateur is offline   Trả Lời Với Trích Dẫn
The Following User Says Thank You to amateur For This Useful Post:
huynhcongbang (12-03-2012)
Old 06-03-2012, 06:24 PM   #73
TBN_146
+Thành Viên+
 
Tham gia ngày: Nov 2011
Đến từ: THPT chuyên Phan Bội Châu - Nghệ An
Bài gởi: 97
Thanks: 27
Thanked 35 Times in 28 Posts
Trích:
Nguyên văn bởi kien10a1 View Post
Bài 23: Cho x,y,z dương tích bằng 1. CMR:
$9+ \frac{1}{x^3}+\frac{1}{y^3}+\frac{1}{z^3}\geq (x+y+z)(xy+yz+zx)+ \frac{(xy+yz+zx)^2}{x+y+z} $
Bài này có thể xử lý bằng Vornicu schur, không biết bạn Kiên có cách khác không? Giả sử $a\geq b\geq c $
BĐT tương đương
$\sum (a-b)(a-c)(\frac{1}{bc}-\frac{1}{ab+bc+ca})\geq 0 $
Chú ý $S_{a},S_{b},S_{c}\geq 0 $ và $S_{a}\geq S_{b} $ ta có đpcm
[RIGHT][I][B]Nguồn: MathScope.ORG[/B][/I][/RIGHT]
 
__________________
crazy
TBN_146 is offline   Trả Lời Với Trích Dẫn
Old 06-03-2012, 08:59 PM   #74
Win-DungDan
+Thành Viên+
 
Tham gia ngày: Nov 2011
Đến từ: Bốn ph­­ưong đều là nhà
Bài gởi: 9
Thanks: 9
Thanked 5 Times in 4 Posts
Trích:
Nguyên văn bởi Win-DungDan View Post
Tôi thấy có bài BDT rất khó của bạn Cẩn trên mathlinks xin đưa cùng các bạn trải nghiệm:
Bài 12: Cho các số thực dương sao cho a+b+c = 1 . Chứng minh rằng:
$\[\frac{36}{{{a}^{2}}b\text{+}{{b}^{2}}c\text{+}{{c} ^{2}}a}\text{+}\frac{1}{abc}\ge 343\] $ .
Sao không bạn nào thử sức bài này vậy nhỉ.. Các bạn cố gắng nha.
[RIGHT][I][B]Nguồn: MathScope.ORG[/B][/I][/RIGHT]
 
__________________
Đứa con của thần gió
Win-DungDan is offline   Trả Lời Với Trích Dẫn
The Following User Says Thank You to Win-DungDan For This Useful Post:
than-dong (07-03-2012)
Old 06-03-2012, 09:23 PM   #75
kien10a1
+Thành Viên+
 
kien10a1's Avatar
 
Tham gia ngày: Feb 2011
Đến từ: Vĩnh Yên- Vĩnh Phúc
Bài gởi: 371
Thanks: 43
Thanked 263 Times in 153 Posts
Gửi tin nhắn qua Yahoo chát tới kien10a1
Trích:
Nguyên văn bởi TBN_146 View Post
Bài này có thể xử lý bằng Vornicu schur, không biết bạn Kiên có cách khác không? Giả sử $a\geq b\geq c $
BĐT tương đương
$\sum (a-b)(a-c)(\frac{1}{bc}-\frac{1}{ab+bc+ca})\geq 0 $
Chú ý $S_{a},S_{b},S_{c}\geq 0 $ và $S_{a}\geq S_{b} $ ta có đpcm
Bài này được biến đổi ra từ USA TST 2009:

$\[ x^{3}(y^{2}+z^{2})^{2}+y^{3}(z^{2}+x^{2})^{2}+z^{3 }(x^{2}+y^{2})^{2}\geq xyz[xy(x+y)^{2}+yz(y+z)^{2}+zx(z+x)^{2}]. \] $(chuẩn hóa xyz=1 là có bài trên)
Đặt $x=1/a,y=1/b,z=1/c $
Bất đẳng thức trên tương đương: $\sum a^{2}(b+c)(a-b)(a-c)\ge 0. $, đúng theo Vornicu schur ( cách của anh Cẩn)
[RIGHT][I][B]Nguồn: MathScope.ORG[/B][/I][/RIGHT]
 
__________________
Quay về với nơi bắt đầu

thay đổi nội dung bởi: kien10a1, 06-03-2012 lúc 09:28 PM
kien10a1 is offline   Trả Lời Với Trích Dẫn
Trả lời Gởi Ðề Tài Mới

Bookmarks


Quuyền Hạn Của Bạn
You may not post new threads
You may not post replies
You may not post attachments
You may not edit your posts

BB code is Mở
Smilies đang Mở
[IMG] đang Mở
HTML đang Tắt

Chuyển đến


Múi giờ GMT. Hiện tại là 05:35 AM.


Powered by: vBulletin Copyright ©2000-2024, Jelsoft Enterprises Ltd.
Inactive Reminders By mathscope.org
[page compression: 120.73 k/137.46 k (12.17%)]